Verfolgen der richtigen Klassen vs. Sets, wenn gezeigt wird, dass ZFC ein zählbares Modell hat (vorausgesetzt, ZFC ist konsistent)

Ich habe die Behauptung gehört, dass ZFC ein zählbares Modell in ZFC hat, wenn ZFC konsistent ist. Soweit ich das beurteilen kann, ist dies eine direkte Folge des Gödelschen Vollständigkeitssatzes und des Löwenheim-Skolem-Satzes.

Ich bin jedoch neugierig, wie vorsichtig man bei der Formulierung des Arguments mit Buchhaltungsdetails in Bezug auf Sets und richtige Klassen sein muss ... oder wenn das Problem aus einem offensichtlichen Grund, den ich vermisse, einfach nicht auftaucht.

Meine Frage ist also zweigeteilt

  • Gibt uns der Vollständigkeitssatz überhaupt ein Modell über einer Menge?
  • Wenn wir zunächst ein Modell einer richtigen Klasse vorziehen, können wir dann Löwenheim-Skolem verwenden?

Wenn ich nicht standardmäßig ein Modell über X sage , meine ich ein Modell, dessen Universum X ist. Ich sage nicht standardmäßig ein Modell in X , wenn die grundlegende Mengenlehre, die zu seiner Konstruktion verwendet wird, die Mengenlehre X ist.

ZFC ist eine Theorie erster Ordnung mit einem Beziehungssymbol , nach Gödels Vollständigkeitssatz hat es ein Modell. Ich bin mir nicht sicher, ob uns dieser Satz die Existenz eines Modells über einer Menge verspricht oder ob das versprochene Modell stattdessen ein Modell über einer echten Klasse sein könnte.

Außerdem hat ZFC seither keine endlichen Modelle , P ( ) , P ( P ( ) ) , sind alle verschieden, wo P bezeichnet den Leistungssatz.

Nehmen wir zum Zwecke der Argumentation an, dass wir schlussfolgern, dass ZFC ein Modell über einer richtigen Klasse hat (unter Verwendung von ZFC als Rohmaterial zum Erstellen des Modells). In diesem Fall interpretieren wir als im ZFC. Wir haben also ein triviales Modell gebaut, nennen wir es M . Das Universum von M ist die Klasse aller Sets in ZFC. Es ist möglich , dass wir durch Anwendung des Vollständigkeitssatzes ein Modell über eine Menge bekommen hätten, wodurch dieses Problem verschwinden würde. Ich bin mir nicht sicher. Angenommen, wir haben uns entschieden zu pflücken M als unser Modell, auch wenn es eine unbequeme Wahl ist.

Der zweite Pfeiler des Arguments ist, soweit ich das beurteilen kann, eine Anwendung des (abwärtsgerichteten) Löwenheim-Skolem-Theorems. Das nach unten gerichtete Löwenheim-Skolem-Theorem befasst sich nicht explizit mit der Grenze der [eigentlichen Klasse], daher frage ich mich, wie das Argument funktioniert. Diese Frage beruft sich auf Löwenheim-Skolem als Erklärung für die Existenz zählbarer Modelle.

In dem im Wikipedia-Artikel skizzierten Beweis des Löwenheim-Skolem-Theorems verwenden wir das Auswahlaxiom wiederholt, um Elemente des Universums auszuwählen, die in das neue Universum für das von uns konstruierte Modell aufgenommen werden müssen .

Eine strenge Lektüre des Auswahlaxioms verspricht nichts über die Fähigkeit, willkürliche Entscheidungen über richtige Klassen zu treffen (und eine Auswahlfunktion kann keine richtige Klasse als Domäne haben), daher bin ich mir nicht sicher, wie ich von meinem trivialen Selbstmodell kommen soll von ZFC M zu jedem Modell, dessen Universum eine Menge ist, geschweige denn eine abzählbare Menge.

Der Vollständigkeitssatz garantiert ein Modell über einer Menge. Im Allgemeinen werden in der Modelltheorie, sofern nicht anders angegeben, alle Modelle als über Mengen betrachtet.

Antworten (1)

Gibt uns der Vollständigkeitssatz überhaupt ein Modell über einer Menge?

Ja. Der Vollständigkeitssatz besagt, dass jede konsistente Theorie ein Modell hat, und in diesem Zusammenhang ist „Modell“ so definiert, dass es ein Modell über einer Menge bedeutet.

Wenn wir zunächst ein Modell einer richtigen Klasse vorziehen, können wir dann Löwenheim-Skolem verwenden?

Dies ist ein subtiles Problem - was bedeutet es überhaupt, ein Modell über einer richtigen Klasse zu haben? Das Problem ist, dass die übliche rekursive Definition dessen, was es bedeutet, dass eine Struktur eine Formel erfüllt, nicht funktioniert, wenn Ihre Struktur eine richtige Klasse ist. Also, wenn Sie eine Struktur haben ( M , ϵ ) Wo M eine richtige Klasse ist, kann man nicht einmal angeben ( M , ϵ ) Z F C in der Sprache von ZFC.

Allerdings für jeden einzelnen Satz φ , können Sie sagen ( M , ϵ ) φ , indem im speziellen Fall von die rekursive Definition "von Hand" durchgeführt wird φ . Wenn Leute also von "Klassenmodellen" sprechen, meinen sie stattdessen normalerweise, dass es ein Theoremschema gibt , das dies für jedes Axiom besagt φ von ZFC, das kannst du beweisen ( M , ϵ ) φ . Alternativ könnten Sie davon ausgehen, dass Ihr Klassenmodell die besondere Eigenschaft hat, dass Sie dafür tatsächlich eine Formelzufriedenheitsrelation definieren können, was in der Praxis jedoch selten der Fall ist.

Was die Durchführung eines Löwenheim-Skolem-Arguments für Klassenmodelle betrifft, können Sie dies tun, solange Sie Zufriedenheit in Bezug auf alle Formeln definieren können, die Ihnen wichtig sind. (Das bedeutet, dass Sie es wie gewohnt ausführen können, wenn Ihr Klassenmodell eine Zufriedenheitsbeziehung hat, oder wenn dies nicht der Fall ist, können Sie es nur ausführen, um ein Untermodell zu erhalten, das endlich viele bestimmte Axiome von ZFC erfüllt, anstatt alle von ZFC .) Das Problem mit dem Auswahlaxiom, auf das Sie hinweisen, kann mit Scotts Trick vermieden werden . Jedes Mal, wenn der Beweis von Löwenheim-Skolem Sie beispielsweise dazu auffordern würde, ein Element mit einer bestimmten Eigenschaft auszuwählen, können Sie, anstatt nur ein Element auszuwählen, alle auswählendie Elemente mit der gewünschten Eigenschaft, die einen minimalen Rang haben. Es kann unzählige solcher Elemente geben, sodass Sie möglicherweise kein zählbares Modell erhalten, aber es wird Sie zumindest auf ein Modell in festgelegter Größe bringen. Dann können Sie gewöhnliches Löwenheim-Skolem verwenden, um zu einem zählbaren Modell zu gelangen.

Beachten Sie, dass Sie in diesem schwachen (Theorem-Schema) Sinne von "Klassenmodell" nicht einmal davon ausgehen müssen, dass ZFC konsistent ist, um ein Klassenmodell von ZFC zu erhalten: ( v , ) ist ein Klassenmodell von ZFC. Das obige Argument zeigt dann, dass Sie für jede endliche Teilmenge der Axiome von ZFC beweisen können, dass es ein (Mengen-)Modell dieser Axiome gibt. Dies ist als Reflexionsprinzip bekannt . (Entscheidend ist jedoch, dass dies ein separates Theorem für jeden endlichen Satz von Axiomen ist und kein einzelnes Theorem mit einer quantifizierten Variablen, die einen beliebigen endlichen Satz von Axiomen darstellt. Daher können Sie es nicht mit Kompaktheit kombinieren, um zu schließen, dass ZFC konsistent ist .)